Linear algebra: solving questions that has 2 systems in it

Click For Summary
The discussion centers on solving two linear systems represented by augmented matrices. The user is confused about how to approach the problem, particularly regarding the redundancy in the second row of equations and the implications for consistency. It is clarified that since the coefficient matrix is the same for both systems, identical row operations can be applied to solve them simultaneously. The augmented matrix should include both systems' constants, allowing for a clear comparison of solutions. Ultimately, understanding the structure of the matrix and the elimination process is key to finding the correct solutions for both systems.
mroldboy
Messages
7
Reaction score
0

Homework Statement





solve the 2 systems

x1 + 2x2 - 2x3 =1 x1 + 2x2 - 2x3 = 9
2x1 + 5x2 +x3 = 9 2x1 + 5x2 +x3 = 9
x1 + 3x2 + 4x3 = 9 x1 + 3x2 + 4x3 = -2

the question gives this explanation

by doing elimination on a 3x5 augmented matrix and the performing two back substitutions.



Homework Equations



I don't think I understand what this question is asking, if someone could explain what it means with its explanation it would help a lot.

This is my first week taking linear algebra as well.


The Attempt at a Solution



I thought it meant to put the two systems together, but since the 2nd row of equations are the same there would only be 5, not 6 I would use so it would be 3x5.

1 2 -2 1
2 5 1 9
1 3 4 9
1 2 -2 9
1 3 4 -2

but then that would make 2 rows
0 0 0 x
and then wouldn't it be inconsistent. But the book has answers to the question so its not that. What is this question even asking me to do? Sorry if didn't put the matrix in the right form for on here.
 
Physics news on Phys.org
The only difference between the two systems is the constants on the righthand side of the equations. Because the coefficient matrix is identical for both systems, you'd find you would apply exactly the same row operations to find each solution, so it makes sense just to reduce both systems at the same time using the matrix

<br /> \left(\begin{array}{ccc|cc}<br /> 1 &amp; 2 &amp; -2 &amp; 1 &amp; 9 \\<br /> 2 &amp; 5 &amp; 1 &amp; 9 &amp; 9 \\<br /> 1 &amp; 3 &amp; 4 &amp; 9 &amp; -2<br /> \end{array}\right)<br />

The fourth column consists of the constants from the first system, and the fifth column consists of the constants from the second system.
 
Question: A clock's minute hand has length 4 and its hour hand has length 3. What is the distance between the tips at the moment when it is increasing most rapidly?(Putnam Exam Question) Answer: Making assumption that both the hands moves at constant angular velocities, the answer is ## \sqrt{7} .## But don't you think this assumption is somewhat doubtful and wrong?

Similar threads

  • · Replies 6 ·
Replies
6
Views
1K
  • · Replies 2 ·
Replies
2
Views
2K
Replies
8
Views
2K
  • · Replies 5 ·
Replies
5
Views
3K
  • · Replies 5 ·
Replies
5
Views
2K
Replies
4
Views
2K
  • · Replies 24 ·
Replies
24
Views
3K
  • · Replies 25 ·
Replies
25
Views
3K
  • · Replies 2 ·
Replies
2
Views
2K
  • · Replies 10 ·
Replies
10
Views
2K